What is the magnitude of r'(t) if r'(t) is <1,2t,3t2>?

  • Thread starter Thread starter ProPatto16
  • Start date Start date
  • Tags Tags
    Magnitude
Click For Summary
The magnitude of r'(t) is calculated as |r'(t)| = √(1 + 4t² + 9t⁴). This expression is derived from the components of the vector r'(t) = <1, 2t, 3t²>. The discussion focuses on finding a suitable denominator for the vector r(t) = <t, t², t³> by dividing r(t) by r'(t). The simplification does not yield a more manageable form, indicating a potentially complex problem. Overall, the challenge lies in the lengthy calculations required for further analysis.
ProPatto16
Messages
323
Reaction score
0

Homework Statement


whats |r'(t)| if r'(t) is <1,2t,3t2>


Homework Equations





The Attempt at a Solution



im finding normal and binormal vectors and i need to divide r(t)/r'(t)

|r'(t)| = sqrt (12+(2t)2+(3t2)2)

how does that simplify down to something that can be an appropriate denominator for r(t)=<t,t2,t3>
 
Physics news on Phys.org
It is
\sqrt{1+ 4t^2+ 9t^4}
but it really does not reduce any more than that.
 
yeah. just looks like its going to be a longwinded ridiculous question
 
Question: A clock's minute hand has length 4 and its hour hand has length 3. What is the distance between the tips at the moment when it is increasing most rapidly?(Putnam Exam Question) Answer: Making assumption that both the hands moves at constant angular velocities, the answer is ## \sqrt{7} .## But don't you think this assumption is somewhat doubtful and wrong?

Similar threads

  • · Replies 20 ·
Replies
20
Views
3K
  • · Replies 7 ·
Replies
7
Views
2K
Replies
1
Views
1K
  • · Replies 1 ·
Replies
1
Views
1K
  • · Replies 2 ·
Replies
2
Views
1K
  • · Replies 22 ·
Replies
22
Views
7K
  • · Replies 3 ·
Replies
3
Views
5K
  • · Replies 2 ·
Replies
2
Views
4K
  • · Replies 3 ·
Replies
3
Views
2K
  • · Replies 1 ·
Replies
1
Views
1K